Wednesday 7 November 2018

integration - Simple, yet evasive integral from zero to $pi/2$




Q: Evaluate$\newcommand{\dx}{\mathrm dx}\newcommand{\du}{\mathrm du}\newcommand{\dv}{\mathrm dv}\newcommand{\dtheta}{\mathrm d\theta}\newcommand{\dw}{\mathrm dw}$$$I=\int\limits_0^{\pi/2}\dx\,\frac {\left(\log\sin x\log\cos x\right)^2}{\sin x\cos x}$$





I'm out of ideas on what to do. I tried using the astute limit identity for integration, but that lead to nowhere, because it's simply the same integral i.e adding the two together, doesn't yield a simplification whatsoever$$I=\int\limits_0^{\pi/2}\dx\,\frac {\log^2\sin x\log^2\cos x}{\sin x\cos x}=\int\limits_0^{\pi/2}\du\,\frac {\log^2\cos u\log^2\sin u}{\cos u\sin u}$$I've looked into making a u-substitution, but have no idea where to begin. I plan to use a trigonometric identity, namely$$\sec x\csc x=\cot x+\tan x$$but I'm not sure what to do afterwards because we get$$I=\int\limits_0^{\pi/2}\dx\,\cot x\log^2\sin x\log^2\cos x+\int\limits_0^{\pi/2}\dx\,\tan x\log^2\sin x\log^2\cos x$$I would appreciate it if you guys gave me an idea on where to begin!



Also, as a side note, I've used \newcommand on \dx,\du,\dv, and \dw to automatically change to $\dx$, $\du$, $\dv$, and $\dw$ respectively, if you guys don't mind! Just to make it easier for the differential operator!


Answer



Change variable to $t = \sin^2 x$ and notice
$$\begin{align}\frac{dx}{\sin x\cos x} &= \frac{\sin x\cos x dx}{\sin^2 x\cos^2 x} = \frac12\frac{dt}{t(1-t)}\\
\log\sin x \log\cos x &= \frac14\log\sin^2 x \log \cos^2 x = \frac14 \log t\log(1-t)
\end{align}

$$
The integral at hand can be rewritten as
$$\mathcal{I} \stackrel{def}{=} \int_0^{\pi/2} \frac{(\log\sin x\log\cos x)^2}{\sin x\cos x} dx = \frac{1}{32}\int_0^1 \frac{\log^2 t \log^2(1-t)}{t(1-t)} dt$$
Since $\displaystyle\frac{1}{t(1-t)} = \frac1t + \frac{1}{1-t}$, by replacing $t$ by $1-t$ in part of the integral, we find



$$\begin{align}
\mathcal{I}
&= \frac{1}{16}\int_0^1 \frac{\log^2 t\log^2(1-t)}{t} dt
= \frac{1}{48}\int_0^1 \log^2(1-t) d\log^3 t\\
&\stackrel{\text{I.by.P}}{=} \frac{1}{48}\left\{

\left[\log^2(1-t)\log^3 t\right]_0^1 + 2
\int_0^1 \log^3 t\frac{\log(1-t)}{1-t}dt\right\}\\
&= -\frac{1}{24}\int_0^1 \frac{\log^3 t}{1-t}\sum_{n=1}^\infty\frac{t^n}{n} dt
= -\frac{1}{24}\int_0^1 \log^3 t\sum_{n=1}^\infty H_nt^n dt\\
&\stackrel{t = e^{-y}}{=} \frac{1}{24} \int_0^\infty y^3 \sum_{n=1}^\infty H_n e^{-(n+1)y} dy
= \frac14 \sum_{n=1}^\infty \frac{H_n}{(n+1)^4}
\end{align}
$$
where $H_n$ are the $n^{th}$ harmonic number. By rearranging its terms, the last sum should be expressible in terms of zeta functions. I'm lazy, I just ask WA to evaluate the sum. As expected, last sum equals to $2\zeta(5) - \frac{\pi^2}{6}\zeta(3)$${}^\color{blue}{[1]}$.




As a consequence, the integral at hand equals to:



$$\mathcal{I}
= \frac{12\zeta(5) - \pi^2\zeta(3)}{24}
\approx 0.024137789997360933616411382857235691008...$$



Notes




  • $\color{blue}{[1]}$ It turns out we can compute this sum using an

    identity by Euler.
    $$2\sum_{n=1}^\infty \frac{H_n}{n^m} = (m+2)\zeta(m+1) - \sum_{n=1}^{m-2}\zeta(m-n)\zeta(n+1),\quad\text{ for } m = 2, 3, \ldots$$
    In particular, for $m = 4$, this identity becomes
    $$\sum_{n=1}^\infty \frac{\zeta(n)}{n^4}
    = 3\zeta(5) - \zeta(2)\zeta(3)$$
    and we can evaluate our sum as
    $$\begin{align}\mathcal{I}
    &= \frac14\sum_{n=1}^{\infty}\frac{H_{n}}{(n+1)^4}
    = \frac14\sum_{n=1}^{\infty}\left(\frac{H_{n+1}}{(n+1)^4}-\frac{1}{(n+1)^5}\right)
    = \frac14\left(\sum_{n=1}^\infty \frac{H_n}{n^4} - \zeta(5)\right)\\

    &= \frac14(2\zeta(5) - \zeta(2)\zeta(3))
    = \frac{12\zeta(5) - \pi^2\zeta(3)}{24}
    \end{align}
    $$


No comments:

Post a Comment

real analysis - How to find $lim_{hrightarrow 0}frac{sin(ha)}{h}$

How to find $\lim_{h\rightarrow 0}\frac{\sin(ha)}{h}$ without lhopital rule? I know when I use lhopital I easy get $$ \lim_{h\rightarrow 0}...